Summer is a great time to explore cool problems to keep your skills sharp!  Schedule a class today!

G
Topic
First Poster
Last Poster
inequality
SunnyEvan   5
N 4 minutes ago by SunnyEvan
Let $ x,y \geq 0 ,$ such that : $ \frac{x^2}{x^3+y}+\frac{y^2}{x+y^3} \geq 1 .$
Prove that : $$ x^2+y^2-xy \leq x+y $$$$ (x+\frac{1}{2})^2+(x+\frac{1}{2})^2 \leq \frac{5}{2} $$$$ (x+1)^2+(y+1)^2 \leq 5 $$$$ (x+2)^2+(y+2)^2 \leq 13 $$
5 replies
SunnyEvan
Yesterday at 1:51 PM
SunnyEvan
4 minutes ago
Bugs Bunny at it again
Rijul saini   8
N an hour ago by quantam13
Source: LMAO 2025 Day 2 Problem 1
Bugs Bunny wants to choose a number $k$ such that every collection of $k$ consecutive positive integers contains an integer whose sum of digits is divisible by $2025$.

Find the smallest positive integer $k$ for which he can do this, or prove that none exist.

Proposed by Saikat Debnath and MV Adhitya
8 replies
Rijul saini
Wednesday at 7:01 PM
quantam13
an hour ago
The Bank of Bath
TelMarin   101
N an hour ago by monval
Source: IMO 2019, problem 5
The Bank of Bath issues coins with an $H$ on one side and a $T$ on the other. Harry has $n$ of these coins arranged in a line from left to right. He repeatedly performs the following operation: if there are exactly $k>0$ coins showing $H$, then he turns over the $k$th coin from the left; otherwise, all coins show $T$ and he stops. For example, if $n=3$ the process starting with the configuration $THT$ would be $THT \to HHT  \to HTT \to TTT$, which stops after three operations.

(a) Show that, for each initial configuration, Harry stops after a finite number of operations.

(b) For each initial configuration $C$, let $L(C)$ be the number of operations before Harry stops. For example, $L(THT) = 3$ and $L(TTT) = 0$. Determine the average value of $L(C)$ over all $2^n$ possible initial configurations $C$.

Proposed by David Altizio, USA
101 replies
TelMarin
Jul 17, 2019
monval
an hour ago
Intersections and concyclic points
Lukaluce   2
N an hour ago by AylyGayypow009
Source: 2025 Junior Macedonian Mathematical Olympiad P2
Let $B_1$ be the foot of the altitude from the vertex $B$ in the acute-angled $\triangle ABC$. Let $D$ be the midpoint of side $AB$, and $O$ be the circumcentre of $\triangle ABC$. Line $B_1D$ meets line $CO$ at $E$. Prove that the points $B, C, B_1$, and $E$ lie on a circle.
2 replies
Lukaluce
May 18, 2025
AylyGayypow009
an hour ago
Calvin needs to cover all squares
Rijul saini   5
N an hour ago by quantam13
Source: India IMOTC 2025 Day 2 Problem 1
Consider a $2025\times 2025$ board where we identify the squares with pairs $(i,j)$ where $i$ and $j$ denote the row and column number of that square, respectively.

Calvin picks two positive integers $a,b<2025$ and places a pawn at the bottom left corner (i.e. on $(1,1)$) and makes the following moves. In his $k$-th move, he moves the pawn from $(i,j)$ to either $(i+a,j)$ or $(i,j+a)$ if $k$ is odd and to either $(i+b,j)$ and $(i,j+b)$ if $k$ is even. Here all the numbers are taken modulo $2025$. Find the number of pairs $(a,b)$ that Calvin could have picked such that he can make moves so that the pawn covers all the squares on the board without being on any square twice.

Proposed by Tejaswi Navilarekallu
5 replies
+1 w
Rijul saini
Wednesday at 6:35 PM
quantam13
an hour ago
Inspired by SunnyEvan
sqing   0
an hour ago
Source: Own
Let $ x,y \geq 0 ,  \frac{x^2}{x^3+y}+\frac{y^2}{x+y^3} \geq 1 .$ Prove that
$$ (x-\frac{1}{2})^2+(y+\frac{1}{2})^2 \leq \frac{5}{2} $$$$ (x-1)^2+(y+1)^2 \leq 5 $$$$ (x-2)^2+(y+2)^2 \leq 13$$$$ (x-\frac{1}{2})^2+(y+1)^2 \leq \frac{17}{4} $$$$ (x-1)^2+(y+2)^2 \leq 10  $$$$ (x-\frac{1}{2})^2+(y+2)^2 \leq \frac{37}{4}  $$
0 replies
sqing
an hour ago
0 replies
Beware the degeneracies!
Rijul saini   8
N an hour ago by AR17296174
Source: India IMOTC 2025 Day 1 Problem 1
Let $a,b,c$ be real numbers satisfying $$\max \{a(b^2+c^2),b(c^2+a^2),c(a^2+b^2) \} \leqslant 2abc+1$$Prove that $$a(b^2+c^2)+b(c^2+a^2)+c(a^2+b^2) \leqslant 6abc+2$$and determine all cases of equality.

Proposed by Shantanu Nene
8 replies
Rijul saini
Wednesday at 6:30 PM
AR17296174
an hour ago
Griphook the globin plays a game
mathscrazy   19
N an hour ago by heheman
Source: INMO 2025/5
Greedy goblin Griphook has a regular $2000$-gon, whose every vertex has a single coin. In a move, he chooses a vertex, removes one coin each from the two adjacent vertices, and adds one coin to the chosen vertex, keeping the remaining coin for himself. He can only make such a move if both adjacent vertices have at least one coin. Griphook stops only when he cannot make any more moves. What is the maximum and minimum number of coins he could have collected?

Proposed by Pranjal Srivastava and Rohan Goyal
19 replies
mathscrazy
Jan 19, 2025
heheman
an hour ago
Inspired by SunnyEvan
sqing   1
N an hour ago by sqing
Source: Own
Let $ x,y \geq 0 ,  \frac{x^2}{x^3+y}+\frac{y^2}{x+y^3} \geq 1 .$ Prove that
$$ |x^k-y^k|+ xy \leq 1  $$Where $ k=1,2,3,4.$
$$ |x^4-y^4|+6xy \leq 6$$$$|x^3-y^3|+270xy \leq 270$$
1 reply
sqing
2 hours ago
sqing
an hour ago
A beautiful Lemoine point problem
phonghatemath   1
N 2 hours ago by phonghatemath
Source: my teacher
Given triangle $ABC$ inscribed in a circle with center $O$. $P$ is any point not on (O). $AP, BP, CP$ intersect $(O)$ at $A', B', C'$. Let $L, L'$ be the Lemoine points of triangle $ABC, A'B'C'$ respectively. Prove that $P, L, L'$ are collinear.
1 reply
phonghatemath
2 hours ago
phonghatemath
2 hours ago
Floor fun...ctional equation
CyclicISLscelesTrapezoid   21
N 2 hours ago by peace09
Source: USA TSTST 2022/8
Let $\mathbb{N}$ denote the set of positive integers. Find all functions $f \colon \mathbb{N} \to \mathbb{Z}$ such that \[\left\lfloor \frac{f(mn)}{n} \right\rfloor=f(m)\]for all positive integers $m,n$.

Merlijn Staps
21 replies
CyclicISLscelesTrapezoid
Jun 27, 2022
peace09
2 hours ago
Inequality with n-gon sides
mihaig   3
N Apr 22, 2025 by mihaig
Source: VL
If $a_1,a_2,\ldots, a_n~(n\geq3)$ are are the lengths of the sides of a $n-$gon such that
$$\sum_{i=1}^{n}{a_i}=1,$$then
$$(n-2)\left[\sum_{i=1}^{n}{\frac{a_i^2}{(1-a_i)^2}}-\frac n{(n-1)^2}\right]\geq(2n-1)\left(\sum_{i=1}^{n}{\frac{a_i}{1-a_i}}-\frac n{n-1}\right)^2.$$
When do we have equality?

(V. Cîrtoaje and L. Giugiuc, 2021)
3 replies
mihaig
Feb 25, 2022
mihaig
Apr 22, 2025
Inequality with n-gon sides
G H J
G H BBookmark kLocked kLocked NReply
Source: VL
The post below has been deleted. Click to close.
This post has been deleted. Click here to see post.
mihaig
7398 posts
#1 • 1 Y
Y by dragonheart6
If $a_1,a_2,\ldots, a_n~(n\geq3)$ are are the lengths of the sides of a $n-$gon such that
$$\sum_{i=1}^{n}{a_i}=1,$$then
$$(n-2)\left[\sum_{i=1}^{n}{\frac{a_i^2}{(1-a_i)^2}}-\frac n{(n-1)^2}\right]\geq(2n-1)\left(\sum_{i=1}^{n}{\frac{a_i}{1-a_i}}-\frac n{n-1}\right)^2.$$
When do we have equality?

(V. Cîrtoaje and L. Giugiuc, 2021)
Z K Y
The post below has been deleted. Click to close.
This post has been deleted. Click here to see post.
mihaig
7398 posts
#2
Y by
Taken from http://matinf.upit.ro/
Z K Y
The post below has been deleted. Click to close.
This post has been deleted. Click here to see post.
mihaig
7398 posts
#3
Y by
Medium...not very difficult
Z K Y
The post below has been deleted. Click to close.
This post has been deleted. Click here to see post.
mihaig
7398 posts
#4
Y by
Old, yet new
Z K Y
N Quick Reply
G
H
=
a